0% found this document useful (0 votes)
112 views5 pages

Sol 4 Ma 1 C Prac 08

This document provides solutions to 10 questions from a homework assignment on vector calculus and multivariable optimization problems. The solutions involve taking derivatives, finding critical points, applying the method of Lagrange multipliers to find extrema subject to constraints, and determining whether critical points are maxima, minima or saddle points. The final question solves an optimization problem to determine the maximum production quantity for a company given production and cost functions.

Uploaded by

張芷瑕
Copyright
© Attribution Non-Commercial (BY-NC)
We take content rights seriously. If you suspect this is your content, claim it here.
Available Formats
Download as PDF, TXT or read online on Scribd
0% found this document useful (0 votes)
112 views5 pages

Sol 4 Ma 1 C Prac 08

This document provides solutions to 10 questions from a homework assignment on vector calculus and multivariable optimization problems. The solutions involve taking derivatives, finding critical points, applying the method of Lagrange multipliers to find extrema subject to constraints, and determining whether critical points are maxima, minima or saddle points. The final question solves an optimization problem to determine the maximum production quantity for a company given production and cost functions.

Uploaded by

張芷瑕
Copyright
© Attribution Non-Commercial (BY-NC)
We take content rights seriously. If you suspect this is your content, claim it here.
Available Formats
Download as PDF, TXT or read online on Scribd
You are on page 1/ 5

HOMEWORK 4 SOLUTIONS

All questions are from Vector Calculus, by Marsden and Tromba Question 1: 3.1.16 Let w = f (x, y ) be a function of two variables, and let x = u + v, y = u v. Show that 2w 2w 2w = . uv x2 y 2 Solution. By the chain rule, w w x w y = + = wx wy . v x v y v Thus, 2w uv = = = i.e., 2w 2w 2w = . uv x2 y 2 Question 2: 3.1.22 (a) : Show that the function g (x, t) = 2 + et sin x satises the heat equation: gt = gxx . [Here g (x, t) represents the temperature in a metal rod at position x and time t.] (b) : Sketch the graph of g for t 0. (Hint: Look at sections by the planes t = 0, t = 1, and t = 2.) (c) : What happens to g (x, t) as t ? Interpret this limit in terms of the behavior of heat in the rod. Solution. (a) : Since g (x, y ) = 2 + et sin x, then gt = et sin x, gx = et cos x, and gxx = et sin x. Therefore, gt = gxx . (b) : The graph of g is shown in Figure 1.
Date : Math 1c Practical, 2008.
1

w = (wx wy ) = wx wy u v u u u wy x wy y wx x wx y + + x u y u x u y u wxx + wxy (wyx + wyy ) = wxx wyy

HOMEWORK 4 SOLUTIONS

t=0 4 t=1 t=2 3

2 4 1 2 0 x 0 0 1 2 t 3 4 4 2

Figure 1. The graph of g at t = 0, 1, and 2. (c) : Note that


t

lim g (x, t) = lim (2 + et sin x) = 2


t

This means that the temperature in the rod at position x tends to be a constant (= 2) as the time t is large enough. Question 3: 3.2.2 Determine the second-order Taylor formula for f (x, y ) = x2 1 + y2 + 1 about x0 = 0, y0 = 0.

Solution. We rst compute the partial derivatives up through second order: fx fxy fxx fyy = = = = 2x , (1 + x2 + y 2 )2 8xy , (1 + x2 + y 2 )3 2 + (1 + x2 + y 2 )2 2 + (1 + x2 + y 2 )2 2y (1 + x2 + y 2 )2 8xy fyx = (1 + x2 + y 2 )3 8x2 (1 + x2 + y 2 )3 8y 2 . (1 + x2 + y 2 )3 fy =

HOMEWORK 4 SOLUTIONS

Next, we evaluate these derivatives at (0, 0), obtaining fx (0, 0) = fy (0, 0) = 0, fxy (0, 0) = fyx (0, 0) = 0 and fxx (0, 0) = fyy (0, 0) = 2. Therefore, the second order Taylor formula is
2 f (h) = h2 1 h2 + R2 (0, h),

where h = (h1 , h2 ) and where R2 (0, h) 0 h as h 0.

Question 4: 3.2.6 Determine the second-order Taylor formula for the function f (x, y ) = e(x1) cos y expanded about the point x0 = 1, y0 = 0. Solution. The ingredients needed in the second-order Taylor formula are computed as follows: fx fy fxx fxy fyy = 2(x 1)e(x1) cos y
2 2 2

= e(x1) sin y = 2e(x1) cos y + 4(x 1)2 e(x1) cos y


2 2 2 2

= 2(x 1)e(x1) sin y = fyx = e(x1) cos y.

Evaluating the function and these derivatives at the point (1, 0) gives f (1, 0) fx (1, 0) fxx (1, 0) fxy (1, 0) fyy (1, 0) = = = = = 1 fy (1, 0) = 0 2 fyx (1, 0) = 0 1. and

Consequently, the second order Taylor formula is 1 2 f (h) = 1 + h2 1 h2 + R2 ((1, 0), h), 2 where h = (h1 , h2 ) and where R2 ((1, 0), h) 0 h as h 0.

Question 5: 3.3.7 Find the critical points for the function f (x, y ) = 3x2 + 2xy + 2x + y 2 + y + 4. and then determine whether they are local maxima, local minima, or saddle points.

HOMEWORK 4 SOLUTIONS

Solution. Here, f = 6x + 2y + 2, x We have f f = 0, =0 x y when x = y = 1/4. Therefore, the only critical point is (1/4, 1/4). Now, 2f 2f 2f x2 (1/4, 1/4) = 6, y 2 (1/4, 1/4) = 2, and xy (1/4, 1/4) = 2, which yields D = 6.2 2 = 10 > 0. Therefore (1/4, 1/4) is a local minimum.
2

f = 2x + 2y + 1. y

Question 6: 3.3.17 Find the local maxima and minima for z = (x2 +3y 2 )e1x
2 2

y 2

(2e)(6e) = 12e > 0, and (0, 0) is a local minimum. 2f 2f 2f x2 (0, 1) = 4, y 2 (0, 1) = 12, and xy (0, 1) = 0, which yields D = (4)(12) = 24 > 0, and (0, 1) are local maxima. 2f 2f 2f x2 (1, 0) = 4, y 2 (1, 0) = 4, and xy (0, 1) = 0, which yields D = (4)(4) = 16 < 0, and (1, 0) are saddle points. Question 7: 3.3.25 Write the number 120 as a sum of three numbers so that the sum of the products taken two at a time is a maximum. Solution. Let the three numbers be x, y, z . Thus, x + y + z = 120, We want to nd the maximum value for S (x, y ) We dierentiate to get S = 2x y + 120, x S = x 2y + 120. y = xy + yz + xz = xy + (x + y )(120 x y ) = x2 xy y 2 + 120x + 120y. z = 120 x y.

2f 2 4 2 2 1x2 y 2 , and y 2 = (3 15y + 6y + x (2y 1))e 2f 2 2 1x2 y 2 . xy = 4(3y + x 4)e 2f 2f 2f (0, 0) Therefore, x2 (0, 0) = 2e, y2 (0, 0) = 6e, and xy 2

Solution. We rst locate the critical points of f (x, y ) = (x2 + 3y 2 )e1x y . 2 2 f (x, y ) = e1x y (2x(1 3y 2 x2 )i + 2y (3 x2 3y 2 )j) Thus, f (x, y ) = 0 if and only if (x, y ) = (0, 0), (0, 1), or (1, 0). To determine whether they are maxima or minima, we need to calculate the second partial derivatives. 2f 4 2 2 2 1x2 y 2 x2 = (1 + 2x 3y + x (6y 5))e

= 0, which yields D =

These vanish when x = y = 40, then z = 120 (x + y ) = 40. Therefore, when x = y = z = 40 is the only critical point. The condition 0 x 120, 0 y 120, 0 z 120 describes a cube in R3 and on the boundary of the cube (either x = 0, x = 120, y = 0, y = 120, z = 0, z = 120), S is zero. Therefore the maximum of S occurs on the interior of this cube, i.e., at a local maximum. Since x = 40, y = 40, z = 40 is the only critical point, it must be a maximum.

HOMEWORK 4 SOLUTIONS

Question 8: 3.4.2 Find the extrema of f (x, y ) = x y subject to the constraint x2 y 2 = 2. Solution. By the method of Lagrange multipliers, we write the constraint as g = 0, where g (x, y ) = x2 y 2 2 and then write the Lagrange multiplier equations as f = g . Thus, we get 1 1 x y 2
2 2

= = =

2x 2y 0.

First of all, the rst two equations imply that x = 0 and y = 0. Hence we can eliminate , giving x = y . From the last equation this would imply that 2 = 0. Hence there are no extrema. Question 9: 3.4.22 Let P be a point on a surface S in R3 dened by the equation f (x, y, z ) = 1, where f is of class C 1 . Suppose that P is a point where the distance from the origin to S is maximized. Show that the vector emanating from the origin and ending at P is perpendicular to S . Solution. We want to maximize the function g (x, y, z ) = x2 + y 2 + z 2 subject to the constraint f (x, y, z ) = 1. Suppose this maximum occurs at P = (x0 , y0 , z0 ), then by the method of Lagrange multipliers we have the equations 2x0 2y0 2z0 = {f (x0 , y0 , z0 )}1 = {f (x0 , y0 , z0 )}2 = {f (x0 , y0 , z0 )}3

where {f (x0 , y0 , z0 )}i denotes the ith component of f (x0 , y0 , z0 ), 1 i 3. If v = (x0 , y0 , z0 ) is the vector from the origin ending at P, then these equations say that v = 2 f (x0 , y0 , z0 ). But f (x0 , y0 , z0 ) is perpendicular to S at P, and since v is a scalar multiple of f (x0 , y0 , z0 ) it is also perpendicular to S at P. Question 10: 3.4.28 A companys production function is Q(x, y ) = xy . The cost of production is C (x, y ) = 2x + 3y . If this company can spend C (x, y ) = 10, what is the maximum quantity that can be produced? Solution. We want to maximize Q subject to the constraint C (x, y ) = 10. Since both x, y 0, this imposes the condition that 0 x 5, 0 y 10/3. Thus, we wish to maximize Q on the line segment 2x +3y = 10, x 0, y 0. If the maximum occurs at an interior point (x0 , y0 ) of this segment, then Q(x0 , y0 ) = C (x0 , y0 ); that is, y0 x0 2x0 + 3y0 = = = 2 3 10.

Thus 6 + 6 = 10, = 5/6, y0 = 5/3, x0 = 5/2, Q(x0 , y0 ) = 25/6. The value of Q at the endpoints of this segment are Q(0, 10 3 ) = 0 = Q(5, 0). Consequently the maximum occurs at (5/2, 5/3) and the maximum value of Q is 25/6.

You might also like